Akademisyenler öncülüğünde matematik/fizik/bilgisayar bilimleri soru cevap platformu
0 beğenilme 0 beğenilmeme
176 kez görüntülendi
$\color{red}{\text{Problem[Lokman GÖKÇE]:}}$ $R>0$ olmak üzere $\displaystyle{ f(R) = \iint_\limits{x^2 + y^2<R^2} \dfrac{x^2 + y^2}{1+x^4 + y^4} dx dy }$ olarak tanımlanıyor. Buna göre $$ \displaystyle{\lim_{R\to \infty} f(R)}$$ ifadesinin (varsa) değeri nedir? Aksi halde, bu limitin ıraksak olduğunu kanıtlayınız.

 

$\color{blue}{\text{Not:}}$ Limitin değerini henüz ben de bilmiyorum. Uygun bir vakitte uğraşabilmemiz için buraya not etmiş olalım.
Lisans Matematik kategorisinde (2.6k puan) tarafından 
tarafından düzenlendi | 176 kez görüntülendi

Integralin yakinsak oldugunu biliyor muyuz? Iraksiyor gibi, en azindan daire yerine kare ile sinirlandirinca oyle.

Yani $\displaystyle{ f(R) = \int_{-\infty}^{\infty}\int_{-\infty}^{\infty}\dfrac{x^2 + y^2}{1+x^4 + y^4} dx dy }$ iraksiyor?

 

Iraksıyor olabilir. Yakınsak olup olmadığını bilmiyorum. Eğer ıraksak ise, ıraksak olduğunu kanıtlayalım.

EK: Biraz daha kurcaladım ve ıraksak oluyor.

1 cevap

0 beğenilme 0 beğenilmeme
En İyi Cevap
$\color{red}{\textbf{Çözüm:}}$ $x=r\cos(\theta)$, $y=r\sin(\theta)$ kutupsal dönüşümü yapılırsa $x^2 + y^2 <R^2$ diski $0\leq \theta < 2\pi $, $0<r < R$ dikdörtgensel bölgesine dönüşür. $x^2 + y^2 = r^2$ dir. Ayrıca, kutupsal dönüşümün Jacobian determinantı $|J|=r$ olduğundan $dxdy = |J|dr d\theta = rdr d\theta$ yazılır. Buna göre,

$ \displaystyle{f(R) = \int_\limits{0}^{2\pi} \int_\limits{0}^{R}\dfrac{r^3 dr d\theta}{1 + r^4(3+ \cos(4\theta))/4} } \tag{1} $

olur. $-1\leq  \cos(4\theta) \leq \color{red}{1}$ olduğundan

$$ \displaystyle{f(R) \geq  \int_\limits{0}^{2\pi} \int_\limits{0}^{R}\dfrac{r^3 dr d\theta}{1 + r^4(3+ \color{red}{1} )/4} } =   \int_\limits{0}^{2\pi} \int_\limits{0}^{R}\dfrac{r^3 dr d\theta}{1 + r^4} = 2\pi \int_\limits{0}^{R}\dfrac{r^3}{1 + r^4}dr \tag{2} $$

elde edilir. $1+r^4 = u$ değişken değiştirmesi yapılırsa $4r^3 dr = du$ olur. Böylece,

$$ \displaystyle{ \int_\limits{0}^{R} \dfrac{r^3}{1 + r^4}dr = \dfrac{1}{4} \int_\limits{u_1}^{u_2}  \dfrac{du}{u}  = \left[ \dfrac{1}{4} \ln(1+r^4) \right]_{0}^{R}  = \dfrac{1}{4}\ln(1+R^4) } \tag{3}$$

buluruz. $(3)$ ifadesini $(2)$ de kullanırsak, $f(R) \geq \dfrac{\pi}{2}\cdot \ln(1+R^4) $ buluruz. Buradan,

$\displaystyle{\lim_{R\to\infty} f(R) \geq \lim_{R\to\infty}\dfrac{\pi}{2}\cdot \ln(1+R^4)} = +\infty$ olup $\displaystyle{\lim_{R\to\infty} f(R) = +\infty}$ elde edilir.

 

 

$\color{blue}{\textbf{Notlar:}}$

Sonsuz limitler ile ilgili eşitsizliklerin bu şekilde yazımı literatürde yeri olan bir yazılış tarzı mıdır, emin olamadım. Gerek görülürse yazılışı iyileştirebilirim.

Soruyu yazarken ilk amaç olarak limitten sonlu bir sayısal bir değer elde edebilelim istemiştim. Bu amaca uygun bir başka soru daha paylaşacağım.
(2.6k puan) tarafından 
tarafından seçilmiş
20,208 soru
21,732 cevap
73,299 yorum
1,905,953 kullanıcı